J & K CET Medical J & K - CET Medical Solved Paper-2002

  • question_answer
    If a gymnast sitting on a rotating stool with his arms outstretched, suddenly lowers his hands :

    A)  the angular velocity decreases

    B)  his moment of inertia decreases

    C)  the angular velocity stays constant

    D)  the angular momentum increases

    Correct Answer: B

    Solution :

                    From law of conservation of angular momentum, when no external torque acts on a body its angular momentum remains conserved. \[\tau =I\omega =cons\tan t\]where\[I\]is moment of inertia\[(=m{{r}^{2}})\]and\[\omega \]the angular velocity. When gymnast suddenly lowers his hands his radius of rotation decreases, hence moment of inertia decreases and angular velocity increases.


You need to login to perform this action.
You will be redirected in 3 sec spinner